Sei sulla pagina 1di 100

DEMO : Purchase from www.A-PDF.

com to remove the watermark

1. (a) The period is T = 4(1.50 µs) = 6.00 µs.

(b) The frequency is the reciprocal of the period:

1 1
f = = = 167
. × 105 Hz.
T 6.00µs

(c) The magnetic energy does not depend on the direction of the current (since UB ∝ i2),
so this will occur after one-half of a period, or 3.00 µs.
2. We find the capacitance from U = 21 Q 2 C :

C=
Q2
=
160c
. × 10−6 C h2

= 9.14 × 10−9 F.
c
2U 2 140 × 10 −6 J h
3. According to U = 21 LI 2 = 21 Q 2 C , the current amplitude is

Q 3.00 × 10−6 C
I= = = 4.52 × 10−2 A .
LC c1.10 × 10 Hhc4.00 × 10 Fh
−3 −6
4. (a) We recall the fact that the period is the reciprocal of the frequency. It is helpful to
refer also to Fig. 31-1. The values of t when plate A will again have maximum positive
charge are multiples of the period:

t A = nT =
n
=
n
f 2.00 × 103 Hz
b
= n 5.00 µs , g
where n = 1, 2, 3, 4, … . The earliest time is (n=1) t A = 5.00 µ s.

(b) We note that it takes t = 21 T for the charge on the other plate to reach its maximum
positive value for the first time (compare steps a and e in Fig. 31-1). This is when plate A
acquires its most negative charge. From that time onward, this situation will repeat once
every period. Consequently,

1 1
t = T + (n −1)T = ( 2n − 1) T =
( 2n −1) = ( 2n −1) = 2n −1 2.50 µ s ,
( )( )
2 2 2f 2 ( 2 ×103 Hz )

where n = 1, 2, 3, 4, … . The earliest time is (n=1) t = 2.50 µ s.

(c) At t = 41 T , the current and the magnetic field in the inductor reach maximum values
for the first time (compare steps a and c in Fig. 31-1). Later this will repeat every half-
period (compare steps c and g in Fig. 31-1). Therefore,

T (n − 1)T T
tL = + = ( 2n − 1) = ( 2n − 1)(1.25 µ s ) ,
4 2 4

where n = 1, 2, 3, 4, … . The earliest time is (n=1) t = 1.25 µ s.


5. (a) All the energy in the circuit resides in the capacitor when it has its maximum
charge. The current is then zero. If Q is the maximum charge on the capacitor, then the
total energy is

U=
Q2
=
c
2.90 × 10−6 C h 2

. × 10−6 J.
= 117
c
2C 2 3.60 × 10 F−6
h
(b) When the capacitor is fully discharged, the current is a maximum and all the energy
resides in the inductor. If I is the maximum current, then U = LI2/2 leads to

I=
2U
=
2 1168
. c
× 10−6 J h
. × 10−3 A .
= 558
−3
L 75 × 10 H
6. (a) The angular frequency is

k F x 8.0 N
ω= = = = 89 rad s .
m m c −13
hb
2.0 × 10 m 0.50 kg g
(b) The period is 1/f and f = ω/2π. Therefore,

2π 2π
T= = = 7.0 × 10−2 s.
ω 89 rad s

(c) From ω = (LC)–1/2, we obtain

1 1
C= = = 2.5 × 10−5 F.
ω L
2
b89 rad sg b5.0 Hg
2
7. (a) The mass m corresponds to the inductance, so m = 1.25 kg.

(b) The spring constant k corresponds to the reciprocal of the capacitance. Since the total
energy is given by U = Q2/2C, where Q is the maximum charge on the capacitor and C is
the capacitance,

C=
Q2
=
c
175 × 10 −6 C h 2

= 2.69 × 10−3 F
c
2U 2 5.70 × 10 J−6
h
and
1
k= = 372 N / m.
2.69 × 10−3 m / N

(c) The maximum displacement corresponds to the maximum charge, so


xmax = 1.75 ×10−4 m.

(d) The maximum speed vmax corresponds to the maximum current. The maximum
current is
Q 175 × 10−6 C
I = Qω = = = 3.02 × 10−3 A.
LC b gc
. H 2.69 × 10 F
125 − 3
h
Consequently, vmax = 3.02 × 10–3 m/s.
d
8. We find the inductance from f = ω / 2 π = 2 π LC i −1
.

1 1
L= = . × 10−5 H.
= 38
2 2
c h c6.7 × 10 Fh
4 π f C 4 π 2 10 × 103 Hz 2 −6
9. The time required is t = T/4, where the period is given by T = 2 π / ω = 2 π LC.
Consequently,
T 2 π LC 2 π
t= = =
b0.050 Hgc4.0 × 10 Fh = 7.0 × 10
−6
−4
s.
4 4 4
10. We apply the loop rule to the entire circuit:

⎛ di q ⎞
ε total = ε L + ε C + ε R +
1 1 1
( ) j ⎝ dt C j ⎟
di q
= ∑ ε L j + ε C j + ε R j = ∑ ⎜ L j + + iR j ⎟ = L + + iR
⎜ dt C
j ⎠
with
1 1
L = ∑ Lj , =∑ , R = ∑ Rj
j C j Cj j

and we require εtotal = 0. This is equivalent to the simple LRC circuit shown in Fig. 31-
27(b).
11. (a) After the switch is thrown to position b the circuit is an LC circuit. The angular
frequency of oscillation is ω = 1/ LC . Consequently,

ω 1 1
f = = = = 275 Hz.
2π 2 π LC 2π c54.0 × 10 Hhc6.20 × 10 Fh
−3 −6

(b) When the switch is thrown, the capacitor is charged to V = 34.0 V and the current is
zero. Thus, the maximum charge on the capacitor is Q = VC = (34.0 V)(6.20 × 10–6 F) =
2.11 × 10–4 C. The current amplitude is

b gc h
I = ωQ = 2 πfQ = 2 π 275 Hz 2.11 × 10−4 C = 0.365 A.
12. The capacitors C1 and C2 can be used in four different ways: (1) C1 only; (2) C2 only;
(3) C1 and C2 in parallel; and (4) C1 and C2 in series.

(a) The smallest oscillation frequency is

1 1
f3 = = = 6.0 × 102 Hz .
b
2 π L C1 + C2 g c hc
. × 10−2 H 2.0 × 10−6 F + 5.0 × 10−6 F
2 π 10 h
(b) The second smallest oscillation frequency is

1 1
f1 = = = 7.1×102 Hz .
2π LC1 2π (1.0 ×10 H )( 5.0 ×10 F )
−2 −6

(c) The second largest oscillation frequency is

1 1
f2 = = = 1.1×103 Hz .
2π LC2 2π (1.0 ×10−2 H )( 2.0 ×10−6 F )

(d) The largest oscillation frequency is

1 1 2.0 ×10−6 F+5.0 ×10−6 F


f4 = = = 1.3 ×103 Hz .
2π LC1C2 / ( C1 + C2 ) 2π (1.0 ×10 H )( 2.0 ×10 F)( 5.0 ×10 F)
−2 −6 −6
13. (a) The maximum charge is Q = CVmax = (1.0 × 10–9 F)(3.0 V) = 3.0 × 10–9 C.

(b) From U = 21 LI 2 = 21 Q 2 / C we get

Q 3.0 × 10−9 C
I= = . × 10−3 A.
= 17
LC c3.0 × 10 Hhc10. × 10 Fh
−3 −9

(c) When the current is at a maximum, the magnetic energy is at a maximum also:

U B,max =
1 2 1
2
c
LI = 3.0 × 10−3 H 17
2
hc
. × 10−3 A h 2
= 4.5 × 10−9 J.
14. The linear relationship between θ (the knob angle in degrees) and frequency f is

FG θ IJ ⇒ θ = 180° FG f − 1IJ
H
f = f0 1+
180° K Hf K
0

where f0 = 2 × 105 Hz. Since f = ω/2π = 1/2π LC , we are able to solve for C in terms of
θ:
1 81
C= 2 2 =
4π Lf 0 (1 + θ /180° ) 400000π (180° + θ )
2 2 2

with SI units understood. After multiplying by 1012 (to convert to picofarads), this is
plotted below:
15. (a) Since the frequency of oscillation f is related to the inductance L and capacitance
C by f = 1 / 2 π LC , the smaller value of C gives the larger value of f. Consequently,
f max = 1 / 2 π LCmin , f min = 1 / 2 π LCmax , and

f max Cmax 365 pF


= = = 6.0.
f min Cmin 10 pF

(b) An additional capacitance C is chosen so the ratio of the frequencies is

. MHz
160
r= = 2.96.
0.54 MHz

Since the additional capacitor is in parallel with the tuning capacitor, its capacitance adds
to that of the tuning capacitor. If C is in picofarads (pF), then

C + 365 pF
= 2.96.
C + 10 pF
The solution for C is

C=
b365 pFg − b2.96g b10 pFg = 36 pF.
2

b2.96g − 12

(c) We solve f = 1 / 2 π LC for L. For the minimum frequency C = 365 pF + 36 pF =


401 pF and f = 0.54 MHz. Thus

1 1
L= = = 2.2 × 10−4 H.
b2πg Cf b2πg c401 × 10 Fhc0.54 × 10 Hzh
2 2 2 −12 6 2
16. For the first circuit ω = (L1C1)–1/2, and for the second one ω = (L2C2)–1/2. When the
two circuits are connected in series, the new frequency is

1 1 1
ω′ = = =
Leq Ceq ( L1 + L2 ) C1C2 / ( C1 + C2 ) ( L1C1C2 + L2C2C1 ) / ( C1 + C2 )
1 1
= =ω,
L1C1 ( C1 + C2 ) / ( C1 + C2 )

where we use ω −1 = L1C1 = L2 C2 .


17. (a) We compare this expression for the current with i = I sin(ωt+φ0). Setting (ωt+φ) =
2500t + 0.680 = π/2, we obtain t = 3.56 × 10–4 s.

(b) Since ω = 2500 rad/s = (LC)–1/2,

1 1
L= = = 2.50 × 10−3 H.
b gc
ω C 2500 rad / s 64.0 × 10 F
2 2 −6
h
(c) The energy is

U=
1 2 1
2 2
c hb
LI = 2.50 × 10 −3 H 160
. A g 2
= 3.20 × 10−3 J.
18. (a) Since the percentage of energy stored in the electric field of the capacitor is
(1 − 75.0%) = 25.0% , then
U E q 2 / 2C
= 2 = 25.0%
U Q / 2C

which leads to q / Q = 0.250 = 0.500.

(b) From
U B Li 2 / 2
= 2 = 75.0%,
U LI / 2

we find i / I = 0.750 = 0.866.


19. (a) The total energy U is the sum of the energies in the inductor and capacitor:

q 2 i 2 L ( 3.80 ×10 C ) ( 9.20 ×10 A ) ( 25.0 ×10 H )


−6 2 −3 2−3

U =U E +U B = + = + = 1.98 ×10−6 J.
2C 2 2 ( 7.80 ×10 F ) −6
2

(b) We solve U = Q2/2C for the maximum charge:

c
Q = 2CU = 2 7.80 × 10−6 F 198 hc
. × 10−6 J = 556 h
. × 10−6 C.

(c) From U = I2L/2, we find the maximum current:

I=
2U
=
c
. × 10−6 J
2 198 h
. × 10−2 A.
= 126
L 25.0 × 10−3 H

(d) If q0 is the charge on the capacitor at time t = 0, then q0 = Q cos φ and

φ = cos−1
FG q IJ = cos FG 380
−1 . × 10 C I
−6

J = ±46.9° .
H QK H 556
. × 10 C K
−6

For φ = +46.9° the charge on the capacitor is decreasing, for φ = –46.9° it is increasing.
To check this, we calculate the derivative of q with respect to time, evaluated for t = 0.
We obtain –ωQ sin φ, which we wish to be positive. Since sin(+46.9°) is positive and
sin(–46.9°) is negative, the correct value for increasing charge is φ = –46.9°.

(e) Now we want the derivative to be negative and sin φ to be positive. Thus, we take
φ = +46.9°.
20. (a) From V = IXC we find ω = I/CV. The period is then T = 2π/ω = 2πCV/I = 46.1 µs.

(b) The maximum energy stored in the capacitor is

1 1
U E = CV 2 = (2.20 × 10−7 F)(0.250 V) 2 = 6.88 × 10−9 J .
2 2

(c) The maximum energy stored in the inductor is also U B = LI 2 / 2 = 6.88 nJ .

(d) We apply Eq. 30-35 as V = L(di/dt)max . We can substitute L = CV2/I2 (combining


what we found in part (a) with Eq. 31-4) into Eq. 30-35 (as written above) and solve for
(di/dt)max . Our result is

⎛ di ⎞ V V I2 (7.50 × 10−3 A) 2
⎜ ⎟ = = = = −7
= 1.02 × 103 A/s .
⎝ dt ⎠ max L CV / I
2 2
CV (2.20 ×10 F)(0.250 V)

1
(e) The derivative of UB = 2 Li2 leads to

dU B 1
= LI 2ω sin ωt cos ωt = LI 2ω sin 2ωt .
dt 2

⎛ dU B ⎞ 1 2 1 1
⎟ = LI ω = IV = (7.50 ×10 A)(0.250 V) = 0.938 mW.
−3
Therefore, ⎜
⎝ dt ⎠ max 2 2 2
21. (a) The charge (as a function of time) is given by q = Q sin ωt , where Q is the
maximum charge on the capacitor and ω is the angular frequency of oscillation. A sine
function was chosen so that q = 0 at time t = 0. The current (as a function of time) is

dq
i= = ωQ cos ωt ,
dt

and at t = 0, it is I = ωQ. Since ω = 1/ LC ,

Q = I LC = 2.00 A b g c3.00 × 10 Hhc2.70 × 10 Fh = 180


−3
. × 10 −6 −4
C.

(b) The energy stored in the capacitor is given by

q 2 Q 2 sin 2 ωt
UE = =
2C 2C
and its rate of change is
dU E Q 2ω sin ωt cos ωt
=
dt C

We use the trigonometric identity cosωt sin ωt = 21 sin 2ωt to write this as b g
dU E ωQ 2
dt
=
2C
sin 2ωt . b g
The greatest rate of change occurs when sin(2ωt) = 1 or 2ωt = π/2 rad. This means

π π π
t= =
4ω 4
LC =
4
( 3.00 ×10 −3
H )( 2.70 ×10−6 F ) = 7.07 ×10−5 s.

(c) Substituting ω = 2π/T and sin(2ωt) = 1 into dUE/dt = (ωQ2/2C) sin(2ωt), we obtain

FG dU IJ 2 πQ 2 πQ 2
H dt K = =
E
.
max 2TC TC

Now T = 2 π LC = 2 π c3.00 × 10 Hhc2.70 × 10 Fh = 5.655 × 10


−3 −6 −4
s, so

FG dU IJ c . × 10−4 C
π 180 h 2

H dt K =
c5.655 × 10 shc2.70 × 10 Fh = 66.7 W.
E
−4 −6
max

We note that this is a positive result, indicating that the energy in the capacitor is indeed
increasing at t = T/8.
22. (a) We use U = 21 LI 2 = 21 Q 2 / C to solve for L:

2
⎛ 1.50V ⎞
2 2 2
1 ⎛ Q ⎞ 1 ⎛ CV ⎞ ⎛V ⎞
L = ⎜ ⎟ = ⎜ max ⎟ = C ⎜ max ⎟ = ( 4.00 ×10−6 F ) ⎜ −3
−3
⎟ = 3.60 ×10 H.
C⎝ I ⎠ C⎝ I ⎠ ⎝ I ⎠ ⎝ 50.0 ×10 A ⎠

(b) Since f = ω/2π, the frequency is

1 1
f = = = 133
. × 103 Hz.
2 π LC 2π c3.60 × 10 Hhc4.00 × 10 Fh
−3 −6

(c) Referring to Fig. 31-1, we see that the required time is one-fourth of a period (where
the period is the reciprocal of the frequency). Consequently,

1 1 1
t= T= = . × 10−4 s.
= 188
4 4 f 4 133 e
. × 10 Hz
3
j
23. The loop rule, for just two devices in the loop, reduces to the statement that the
magnitude of the voltage across one of them must equal the magnitude of the voltage
across the other. Consider that the capacitor has charge q and a voltage (which we’ll
consider positive in this discussion) V = q/C. Consider at this moment that the current in
the inductor at this moment is directed in such a way that the capacitor charge is
increasing (so i = +dq/dt). Eq. 30-35 then produces a positive result equal to the V across
the capacitor: V = −L(di/dt), and we interpret the fact that −di/dt > 0 in this discussion to
mean that d(dq/dt)/dt = d2q/dt2 < 0 represents a “deceleration” of the charge-buildup
process on the capacitor (since it is approaching its maximum value of charge). In this
way we can “check” the signs in Eq. 31-11 (which states q/C = − L d2q/dt2) to make sure
we have implemented the loop rule correctly.
24. The assumption stated at the end of the problem is equivalent to setting φ = 0 in Eq.
2
31-25. Since the maximum energy in the capacitor (each cycle) is given by qmax / 2C ,
where qmax is the maximum charge (during a given cycle), then we seek the time for
which
2
qmax 1 Q2 Q
= ⇒ qmax = .
2C 2 2C 2

Now qmax (referred to as the exponentially decaying amplitude in §31-5) is related to Q


(and the other parameters of the circuit) by

qmax = Qe − Rt / 2 L ⇒ ln
FG q IJ = − Rt .
H Q K 2L
max

Setting qmax = Q / 2 , we solve for t:

t=−
2L FG IJ
q
ln max = −
2L FG 1 IJ = L ln 2 .
R H K
Q R
ln
H 2K R
The identities ln (1 / 2 ) = − ln 2 = − 21 ln 2 were used to obtain the final form of the
result.
25. Since ω ≈ ω', we may write T = 2π/ω as the period and ω = 1/ LC as the angular
frequency. The time required for 50 cycles (with 3 significant figures understood) is

⎛ 2π ⎞
(
t = 50T = 50 ⎜ ⎟ = 50 2π LC = 50 2π
⎝ω ⎠
) ( ( 220 ×10 H )(12.0 ×10 F) )
−3 −6

= 0.5104s.

The maximum charge on the capacitor decays according to q max = Qe − Rt / 2 L (this is called
the exponentially decaying amplitude in §31-5), where Q is the charge at time t = 0 (if we
take φ = 0 in Eq. 31-25). Dividing by Q and taking the natural logarithm of both sides, we
obtain
qFG IJ
ln max = −
Rt
QH K 2L
which leads to

2 L ⎛ qmax ⎞ 2 ( 220 × 10−3 H )


R=− ln ⎜ ⎟ = − ln ( 0.99 ) = 8.66 × 10−3 Ω .
t ⎝ Q ⎠ 0.5104s
26. The charge q after N cycles is obtained by substituting t = NT = 2πN/ω' into Eq.
31-25:

q = Qe − Rt / 2 L cos (ω ′t + φ ) = Qe− RNT / 2 L cos ⎡⎣ω ′ ( 2πN / ω ′ ) + φ ⎤⎦


( )
cos ( 2πN + φ )
− RN 2 π L / C / 2 L
= Qe
= Qe − N πR C/L
cos φ .

We note that the initial charge (setting N = 0 in the above expression) is q0 = Q cos φ,
where q0 = 6.2 µC is given (with 3 significant figures understood). Consequently, we
write the above result as qN = q0 exp − N πR C / L . ( )
(
(a) For N = 5, q5 = ( 6.2 µ C ) exp −5π ( 7.2Ω ) 0.0000032F/12H = 5.85 µ C. )
(
(b) For N = 10, q10 = ( 6.2 µ C ) exp −10π ( 7.2Ω ) 0.0000032F/12H = 5.52 µ C. )
(
(c) For N = 100, q100 = ( 6.2 µ C ) exp −100π ( 7.2Ω ) 0.0000032F/12H = 1.93 µ C. )
27. Let t be a time at which the capacitor is fully charged in some cycle and let qmax 1 be
the charge on the capacitor then. The energy in the capacitor at that time is

2
qmax Q 2 − Rt / L
U (t ) = 1
= e
2C 2C

where
qmax1 = Qe − Rt / 2 L

(see the discussion of the exponentially decaying amplitude in §31-5). One period later
the charge on the fully charged capacitor is


qmax 2 = Qe − R (t +T )2 / L where T = ,
ω'
and the energy is
2
qmax Q 2 − R ( t +T ) / L
U (t + T ) = 2
= e .
2C 2C

The fractional loss in energy is

| ∆U | U (t ) − U (t + T ) e − Rt / L − e − R ( t + T )/ L
= = = 1 − e − RT / L .
U U (t ) e − Rt / L

Assuming that RT/L is very small compared to 1 (which would be the case if the
resistance is small), we expand the exponential (see Appendix E). The first few terms are:

− RT / L RT R 2 T 2
e ≈ 1− + + .
L 2 L2

If we approximate ω ≈ ω', then we can write T as 2π/ω. As a result, we obtain

| ∆U | ⎛ RT ⎞ RT 2πR
≈ 1 − ⎜1 − + ⎟≈ = .
U ⎝ L ⎠ L ωL
28. (a) The current through the resistor is

εm 30.0 V
I= = = 0.600 A .
R 50.0 Ω

(b) Regardless of the frequency of the generator, the current is the same, I = 0.600 A .
29. (a) The inductive reactance for angular frequency ωd is given by X L = ωd L , and the
capacitive reactance is given by XC = 1/ωdC. The two reactances are equal if ωdL = 1/ωdC,
or ω d = 1/ LC . The frequency is

ωd 1 1
fd = = = = 6.5 × 102 Hz.
2π 2π LC 2π (6.0×10 H)(10 ×10 F)
−3 −6

(b) The inductive reactance is

XL = ωdL = 2πfdL = 2π(650 Hz)(6.0 × 10–3 H) = 24 Ω.

The capacitive reactance has the same value at this frequency.

(c) The natural frequency for free LC oscillations is f = ω / 2π = 1/ 2π LC , the same as


we found in part (a).
30. (a) We use I = ε/Xc = ωdCε:

I = ωd Cε m = 2πf d Cε m = 2π(1.00×103 Hz)(1.50 × 10−6 F)(30.0 V) = 0.283 A .

(b) I = 2π(8.00 × 103 Hz)(1.50 × 10–6 F)(30.0 V) = 2.26 A.


31. (a) The current amplitude I is given by I = VL/XL, where XL = ωdL = 2πfdL. Since the
circuit contains only the inductor and a sinusoidal generator, VL = εm. Therefore,

VL ε 30.0V
I= = m = = 0.0955A = 95.5 mA.
X L 2πf d L 2π(1.00×103 Hz)(50.0 ×10−3 H)

(b) The frequency is now eight times larger than in part (a), so the inductive reactance XL
is eight times larger and the current is one-eighth as much. The current is now

I = (0.0955 A)/8 = 0.0119 A = 11.9 mA.


32. (a) The circuit consists of one generator across one capacitor; therefore, εm = VC.
Consequently, the current amplitude is

εm
I= = ω Cε m = (377 rad / s)(4.15 × 10−6 F)(25.0 V) = 3.91 × 10−2 A .
XC

(b) When the current is at a maximum, the charge on the capacitor is changing at its
largest rate. This happens not when it is fully charged (±qmax), but rather as it passes
through the (momentary) states of being uncharged (q = 0). Since q = CV, then the
voltage across the capacitor (and at the generator, by the loop rule) is zero when the
current is at a maximum. Stated more precisely, the time-dependent emf ε(t) and current
i(t) have a φ = –90° phase relation, implying ε(t) = 0 when i(t) = I. The fact that φ = –90°
= –π/2 rad is used in part (c).

(c) Consider Eq. 32-28 with ε = − 21 ε m . In order to satisfy this equation, we require
sin(ωdt) = –1/2. Now we note that the problem states that ε is increasing in magnitude,
which (since it is already negative) means that it is becoming more negative. Thus,
differentiating Eq. 32-28 with respect to time (and demanding the result be negative) we
must also require cos(ωdt) < 0. These conditions imply that ωt must equal (2nπ – 5π/6) [n
= integer]. Consequently, Eq. 31-29 yields (for all values of n)

⎛ 5π π ⎞ ⎛ 3⎞
i = I sin ⎜ 2nπ − + ⎟ = (3.91× 10−3 A) ⎜⎜ − −2
⎟⎟ = −3.38 × 10 A,
⎝ 6 2 ⎠ ⎝ 2 ⎠

or | i | = 3.38 ×10−2 A.
33. (a) The generator emf is a maximum when sin(ωdt – π/4) = 1 or

ωdt – π/4 = (π/2) ± 2nπ [n = integer].

The first time this occurs after t = 0 is when ωdt – π/4 = π/2 (that is, n = 0). Therefore,

3π 3π
t= = = 6.73 × 10−3 s .
4ω d 4(350 rad / s)

(b) The current is a maximum when sin(ωdt – 3π/4) = 1, or

ωdt – 3π/4 = (π/2) ± 2nπ [n = integer].

The first time this occurs after t = 0 is when ωdt – 3π/4 = π/2 (as in part (a), n = 0).
Therefore,
5π 5π
t= = . × 10−2 s .
= 112
4ω d 4(350 rad / s)

(c) The current lags the emf by +π / 2 rad, so the circuit element must be an inductor.

(d) The current amplitude I is related to the voltage amplitude VL by VL = IXL, where XL is
the inductive reactance, given by XL = ωdL. Furthermore, since there is only one element
in the circuit, the amplitude of the potential difference across the element must be the
same as the amplitude of the generator emf: VL = εm. Thus, εm = IωdL and

εm 30.0V
L= = = 0138
. H.
Iω d (620 × 10 A)(350 rad / s)
−3
34. (a) The circuit consists of one generator across one inductor; therefore, εm = VL. The
current amplitude is

εm εm 25.0 V
I= = = = 5.22 ×10−3 A .
XL ω d L (377 rad/s)(12.7 H)

(b) When the current is at a maximum, its derivative is zero. Thus, Eq. 30-35 gives εL = 0
at that instant. Stated another way, since ε(t) and i(t) have a 90° phase difference, then ε(t)
must be zero when i(t) = I. The fact that φ = 90° = π/2 rad is used in part (c).

(c) Consider Eq. 31-28 with ε = −ε m / 2 . In order to satisfy this equation, we require
sin(ωdt) = –1/2. Now we note that the problem states that ε is increasing in magnitude,
which (since it is already negative) means that it is becoming more negative. Thus,
differentiating Eq. 31-28 with respect to time (and demanding the result be negative) we
must also require cos(ωdt) < 0. These conditions imply that ωt must equal (2nπ – 5π/6) [n
= integer]. Consequently, Eq. 31-29 yields (for all values of n)

FG 5π π IJ F I
H
i = I sin 2nπ − −
6 2
= (5.22 × 10−3 A)
K 2GH JK
3
= 4.51 × 10−3 A .
35. (a) Now XL = 0, while R = 200 Ω and XC = 1/2πfdC = 177 Ω. Therefore, the
impedance is
Z = R 2 + X C2 = (200 Ω) 2 + (177 Ω) 2 = 267 Ω.

(b) The phase angle is

⎛ X L − XC ⎞ −1 ⎛ 0 − 177 Ω ⎞
φ =tan −1 ⎜ ⎟ = tan ⎜ ⎟ = −41.5°
⎝ R ⎠ ⎝ 200 Ω ⎠

(c) The current amplitude is

εm 36.0 V
I= = = 0.135 A .
Z 267 Ω

(d) We first find the voltage amplitudes across the


circuit elements:

VR = IR = (0.135A)(200Ω) ≈ 27.0V
VC = IX C = (0.135A)(177 Ω) ≈ 23.9V

The circuit is capacitive, so I leads ε m . The phasor


diagram is drawn to scale on the right.
36. (a) The graph shows that the resonance angular frequency is 25000 rad/s, which
means (using Eq. 31-4)

C = (ω2L)−1 = [(25000)2 ×200 × 10−6]−1 = 8.0 µF.

(b) The graph also shows that the current amplitude at resonance is 4.0 A, but at
resonance the impedance Z becomes purely resistive (Z = R) so that we can divide the
emf amplitude by the current amplitude at resonance to find R: 8.0/4.0 = 2.0 Ω.
37. (a) Now XC = 0, while R = 200 Ω and XL = ωL = 2πfdL = 86.7 Ω remain unchanged.
Therefore, the impedance is

Z = R 2 + X L2 = (200 Ω) 2 + (86.7 Ω) 2 = 218 Ω .

(b) The phase angle is, from Eq. 31-65,

⎛ X L − XC ⎞ −1 ⎛ 86.7 Ω − 0 ⎞
φ = tan −1 ⎜ ⎟ = tan ⎜ ⎟ = 23.4° .
⎝ R ⎠ ⎝ 200 Ω ⎠

(c) The current amplitude is now found to be

εm 36.0 V
I= = = 0.165 A .
Z 218 Ω

(d) We first find the voltage amplitudes across the circuit elements:

VR = IR = (0.165 A)(200 Ω) ≈ 33V


VL = IX L = (0.165A)(86.7Ω) ≈ 14.3V

This is an inductive circuit, so εm leads I. The phasor diagram is drawn to scale below.
38. (a) Since Z = R2 + XL2 and XL = ωd L, then as ωd → 0 we find Z → R = 40 Ω.

(b) L = XL /ωd = slope = 60 mH.


39. (a) The capacitive reactance is

1 1 1
XC = = = = 37.9 Ω .
ω d C 2πf d C 2π(60.0 Ηz)(70.0 ×10−6 F)

The inductive reactance 86.7 Ω is unchanged. The new impedance is

Z = R 2 + ( X L − X C ) 2 = (200 Ω) 2 + (37.9 Ω − 86.7 Ω) 2 = 206 Ω .

(b) The phase angle is

⎛ X L − XC ⎞ −1 ⎛ 86.7 Ω − 37.9 Ω ⎞
φ = tan −1 ⎜ ⎟ = tan ⎜ ⎟ = 13.7° .
⎝ R ⎠ ⎝ 200 Ω ⎠

(c) The current amplitude is


εm 36.0 V
I= = = 0.175A.
Z 206 Ω

(d) We first find the voltage amplitudes across the circuit elements:

VR = IR = (0.175 A)(200 Ω) = 35.0 V


VL = IX L = (0.175 A)(86.7 Ω) = 15.2 V
VC = IX C = (0.175 A)(37.9 Ω) = 6.62 V

Note that X L > X C , so that ε m leads I. The phasor diagram is drawn to scale below.
40. (a) The circuit has a resistor and a capacitor (but no inductor). Since the capacitive
reactance decreases with frequency, then the asymptotic value of Z must be the resistance:
R = 500 Ω.

(b) We describe three methods here (each using information from different points on the
graph):

method 1: At ωd = 50 rad/s, we have Z ≈ 700 Ω which gives C = (ωd Z2 - R2 )−1 = 41 µF.

method 2: At ωd = 50 rad/s, we have XC ≈ 500 Ω which gives C = (ωd XC)−1 = 40 µF.

method 3: At ωd = 250 rad/s, we have XC ≈ 100 Ω which gives C = (ωd XC)−1 = 40 µF.
41. The rms current in the motor is

ε rms ε rms 420 V


I rms = = = = 7.61A.
Z R +X ( 45.0 Ω ) + ( 32.0 Ω )
2 2 2 2
L
42. A phasor diagram very much like Fig. 31-11(d) leads to the condition:

VL – VC = (6.00 V)sin(30º) = 3.00 V.

With the magnitude of the capacitor voltage at 5.00 V, this gives a inductor voltage
magnitude equal to 8.00 V. Since the capacitor and inductor voltage phasors are 180° out
of phase, the potential difference across the inductor is −8.00 V .
43. The resistance of the coil is related to the reactances and the phase constant by Eq.
31-65. Thus,

X L − X C ω d L − 1/ ω d C
= = tan φ ,
R R
which we solve for R:

1 ⎛ 1 ⎞ 1 ⎡ 1 ⎤
R= ⎜ ωd L − ⎟= ⎢ (2π)(930 Hz(8.8 ×10−2 H) − −6 ⎥
tan φ ⎝ ω d C ⎠ tan 75° ⎣ (2π)(930 Hz)(0.94 ×10 F ⎦
= 89 Ω .
44. (a) A sketch of the phasors would be very much like Fig. 31-9(c) but with the label
“IC” on the green arrow replaced with “VR.”

(b) We have I R = I XC, or


1
I R = I XC → R =
ωd C

ωd 1 1
which yields f = = = = 159 Hz .
2π 2π RC 2π (50.0 Ω)(2.00 ×10−5 F)

(c) φ = tan−1(−VC /VR) = – 45°.

(d) ωd = 1/RC =1.00 ×103 rad/s.

(e) I = (12 V)/ R2 + XC2 = 6/(25 2) ≈170 mA.


45. (a) For a given amplitude εm of the generator emf, the current amplitude is given by

εm εm
I= = .
Z R + (ω d L − 1/ ω d C ) 2
2

We find the maximum by setting the derivative with respect to ω d equal to zero:

dI
= − ( E ) m [ R 2 + (ω d L − 1 / ω d C ) 2 ]−3/ 2 ω d L −
1 LM OP LM L + 1 OP .
dω d ω dC N Q N ω CQ2
d

The only factor that can equal zero is ω d L − (1 / ω d C ) ; it does so for ω d = 1/ LC = ω .


For this
1 1
ωd = = = 224 rad / s .
LC . H)(20.0 × 10−6 F)
(100

(b) When ω d = ω , the impedance is Z = R, and the current amplitude is

εm 30.0 V
I= = = 6.00 A .
R 5.00 Ω

(c) We want to find the (positive) values of ω d for which I = ε m / 2 R :

εm εm
= .
R + (ω d L − 1/ ω d C )
2 2 2R

This may be rearranged to yield

FGω dL−
1 IJ 2

= 3R 2 .
H ω dC K
Taking the square root of both sides (acknowledging the two ± roots) and multiplying by
ω d C , we obtain
ω 2d ( LC ) ± ω d d i
3CR − 1 = 0 .

Using the quadratic formula, we find the smallest positive solution


− 3CR + 3C 2 R 2 + 4 LC − 3(20.0 ×10−6 F)(5.00 Ω)
ω2 = =
2 LC 2(1.00 H)(20.0 ×10−6 F)
3(20.0 ×10−6 F) 2 (5.00 Ω) 2 + 4(1.00 H)(20.0 ×10 −6 F)
+
2(1.00 H)(20.0 ×10−6 F)
= 219 rad/s .

(d) The largest positive solution

+ 3CR + 3C 2 R 2 + 4 LC + 3(20.0 ×10−6 F)(5.00 Ω)


ω1 = =
2 LC 2(1.00 H)(20.0 ×10−6 F)
3(20.0 × 10−6 F) 2 (5.00 Ω) 2 + 4(1.00 H)(20.0 ×10 −6 F)
+
2(1.00 H)(20.0 ×10−6 F)
= 228 rad/s .

(e) The fractional width is

ω1 − ω 2 228rad/s − 219rad/s
= = 0.040.
ω0 224 rad/s
46. (a) With both switches closed (which effectively removes the resistor from the
circuit), the impedance is just equal to the (net) reactance and is equal to

Xnet = (12 V)/(0.447 A) = 26.85 Ω.

With switch 1 closed but switch 2 open, we have the same (net) reactance as just
discussed, but now the resistor is part of the circuit; using Eq. 31-65 we find

X net 26.85 Ω
R= = = 100 Ω .
tanφ tan15°

(b) For the first situation described in the problem (both switches open) we can reverse
our reasoning of part (a) and find

Xnet first = R tan φ ′ = (100 Ω) tan(–30.9º) = –59.96 Ω.

We observe that the effect of switch 1 implies

XC = Xnet – Xnet firsrt = 26.85 Ω – (–59.96 Ω) = 86.81 Ω.

Then Eq. 31-39 leads to C = 1/ωXC = 30.6 µF.

(c) Since Xnet = XL – XC , then we find L = XL/ω = 301 mH .


47. (a) Yes, the voltage amplitude across the inductor can be much larger than the
amplitude of the generator emf.

(b) The amplitude of the voltage across the inductor in an RLC series circuit is given by
VL = IX L = Iω d L . At resonance, the driving angular frequency equals the natural angular
frequency: ω d = ω = 1/ LC . For the given circuit

L 1.0 H
XL = = = 1000 Ω .
LC (1.0 H)(1.0 ×10−6 F)

At resonance the capacitive reactance has this same value, and the impedance reduces
simply: Z = R. Consequently,

εm εm 10 V
I= = = = 1.0 A .
Z resonance R 10 Ω

The voltage amplitude across the inductor is therefore

VL = IX L = (1.0 A)(1000 Ω) = 1.0 × 103 V

which is much larger than the amplitude of the generator emf.


48. (a) A sketch of the phasors would be very much like Fig. 31-10(c) but with the label
“IL” on the green arrow replaced with “VR.”

(b) We have VR = VL, which implies

I R = I XL → R = ωd L

which yields f = ωd/2π = R/2πL = 318 Hz.

(c) φ = tan−1(VL /VR) = +45°.

(d) ωd = R/L = 2.00×103 rad/s.

(e) I = (6 V)/ R2 + XL2 = 3/(40 2) ≈ 53.0 mA.


49. We use the expressions found in Problem 31-45:

+ 3CR + 3C 2 R 2 + 4 LC − 3CR + 3C 2 R 2 + 4 LC
ω1 = , ω2 = .
2 LC 2 LC

We also use Eq. 31-4. Thus,

∆ω d ω 1 − ω 2 2 3CR LC 3C
= = =R .
ω ω 2 LC L

For the data of Problem 31-45,

∆ω d
ω
b
= 5.00 Ω g c . H
100
h = 387
3 20.0 × 10−6 F
. × 10 −2
.

This is in agreement with the result of Problem 31-45. The method of Problem 31-45,
however, gives only one significant figure since two numbers close in value are
subtracted (ω1 – ω2). Here the subtraction is done algebraically, and three significant
figures are obtained.
50. (a) The capacitive reactance is

1 1
XC = = = 16.6 Ω .
2πfC 2π(400 Hz)(24.0 × 10−6 F)

(b) The impedance is

Z = R 2 + ( X L − X C ) 2 = R 2 + (2πfL − X C ) 2
= (220 Ω) 2 + [2π(400 Hz)(150 ×10−3 H) − 16.6 Ω]2 = 422 Ω .

(c) The current amplitude is

εm 220 V
I= = = 0.521 A .
Z 422 Ω

(d) Now X C ∝ Ceq−1 . Thus, XC increases as Ceq decreases.

(e) Now Ceq = C/2, and the new impedance is

Z = (220 Ω) 2 + [2π(400 Hz)(150 × 10−3 H) − 2(16.6 Ω)]2 = 408 Ω < 422 Ω .

Therefore, the impedance decreases.

(f) Since I ∝ Z −1 , it increases.


51. (a) Since Leq = L1 + L2 and Ceq = C1 + C2 + C3 for the circuit, the resonant frequency
is
1 1
ω= =
b gb
2 π Leq Ceq 2 π L1 + L2 C1 + C2 + C3 g
1
=
c hc
. × 10 H + 2.30 × 10 H 4.00 × 10−6 F + 2.50 × 10−6 F + 3.50 × 10−6 F
2 π 170 −3 −3
h
= 796 Hz.

(b) The resonant frequency does not depend on R so it will not change as R increases.

(c) Since ω ∝ (L1 + L2)–1/2, it will decrease as L1 increases.

(d) Since ω ∝ Ceq−1/2 and Ceq decreases as C3 is removed, ω will increase.


52. The amplitude (peak) value is

b g
Vmax = 2Vrms = 2 100 V = 141 V.
53. The average power dissipated in resistance R when the current is alternating is given
by Pavg = I rms
2
R, where Irms is the root-mean-square current. Since I rms = I / 2 , where I is
the current amplitude, this can be written Pavg = I2R/2. The power dissipated in the same
resistor when the current id is direct is given by P = id2 R. Setting the two powers equal to
each other and solving, we obtain

I 2.60 A
id = = = 184
. A.
2 2
54. Since the impedance of the voltmeter is large, it will not affect the impedance of the
circuit when connected in parallel with the circuit. So the reading will be 100 V in all
three cases.
55. (a) Using Eq. 31-61, the impedance is

Z= (12.0 Ω ) + (1.30 Ω − 0 ) = 12.1 Ω.


2 2

(b) The average rate at which energy has been supplied is

(120 V ) (12.0 Ω ) = 1.186 ×103 W ≈ 1.19 ×103 W.


2
ε rms
2
R
Pavg = =
(12.07 Ω )
2
Z2
56. This circuit contains no reactances, so εrms = IrmsRtotal. Using Eq. 31-71, we find the
average dissipated power in resistor R is

F ε IJ
R=G
2

PR = I
H r + RK
2 m
rms R.

In order to maximize PR we set the derivative equal to zero:

dPR ε m ⎣( r + R ) − 2 ( r + R ) R ⎦ ε m ( r − R )
2 ⎡ 2
⎤ 2

= = =0 ⇒ R=r
(r + R) (r + R)
4 3
dR
57. (a) The power factor is cos φ, where φ is the phase constant defined by the expression
i = I sin(ωt – φ). Thus, φ = – 42.0° and cos φ = cos(– 42.0°) = 0.743.

(b) Since φ < 0, ωt – φ > ωt. The current leads the emf.

(c) The phase constant is related to the reactance difference by tan φ = (XL – XC)/R. We
have
tan φ = tan(– 42.0°) = –0.900,

a negative number. Therefore, XL – XC is negative, which leads to XC > XL. The circuit in
the box is predominantly capacitive.

(d) If the circuit were in resonance XL would be the same as XC, tan φ would be zero, and
φ would be zero. Since φ is not zero, we conclude the circuit is not in resonance.

(e) Since tan φ is negative and finite, neither the capacitive reactance nor the resistance
are zero. This means the box must contain a capacitor and a resistor.

(f) The inductive reactance may be zero, so there need not be an inductor.

(g) Yes, there is a resistor.

(h) The average power is

Pavg =
1
2
b 1
gb
ε m I cos φ = 75.0 V 120
2
gb g
. A 0.743 = 33.4 W.

(i) The answers above depend on the frequency only through the phase constant φ, which
is given. If values were given for R, L and C then the value of the frequency would also
be needed to compute the power factor.
58. The current in the circuit satisfies i(t) = I sin(ωdt – φ), where
εm εm
I= =
Z R 2 + (ω d L − 1/ ω d C )
2

45.0 V
=
(16.0 Ω ) { }
+ ( 3000 rad/s )( 9.20 mH ) − 1/ ⎡⎣( 3000 rad/s )( 31.2 µ F ) ⎤⎦
2 2

= 1.93A

and
⎛ X L − XC ⎞ −1 ⎛ ω d L − 1/ ω d C ⎞
φ = tan −1 ⎜ ⎟ = tan ⎜ ⎟
⎝ R ⎠ ⎝ R ⎠
⎡ ( 3000 rad/s )( 9.20 mH ) 1 ⎤
= tan −1 ⎢ − ⎥
⎣ 16.0 Ω ( 3000 rad/s )(16.0 Ω )( 31.2 µ F ) ⎦
= 46.5°.

(a) The power supplied by the generator is

Pg = i (t )ε (t ) = I sin (ωd t − φ ) ε m sin ωd t


= (1.93A )( 45.0 V ) sin ⎡⎣( 3000 rad/s )( 0.442 ms ) ⎤⎦ sin ⎡⎣( 3000 rad/s )( 0.442 ms ) − 46.5°⎤⎦
= 41.4 W.

(b) With
vc (t ) = Vc sin(ωd t − φ − π / 2) = −Vc cos(ωd t − φ )

where Vc = I / ωd C , the rate at which the energy in the capacitor changes is

d ⎛ q2 ⎞ q
Pc = ⎜ ⎟ = i = ivc
dt ⎝ 2C ⎠ C
⎛ I ⎞ I2
= − I sin (ωd t − φ ) ⎜ ⎟ cos ( d )
ω t − φ = − sin ⎡⎣ 2 (ωd t − φ ) ⎤⎦
⎝ ω d C ⎠ 2ω d C
(1.93A )
2

=− sin ⎡⎣ 2 ( 3000 rad/s )( 0.442 ms ) − 2 ( 46.5° ) ⎤⎦


2 ( 3000 rad/s ) ( 31.2 × 10−6 F )
= −17.0 W.

(c) The rate at which the energy in the inductor changes is


d ⎛1 2⎞ di d 1
PL = ⎜ Li ⎟ = Li = LI sin (ω d t − φ ) ⎡⎣ I sin (ω d t − φ ) ⎤⎦ = ω d LI sin ⎡⎣ 2 (ω d t − φ ) ⎤⎦
2

dt ⎝ 2 ⎠ dt dt 2
1
= ( 3000 rad/s )(1.93A ) ( 9.20 mH ) sin ⎡⎣ 2 ( 3000 rad/s )( 0.442 ms ) − 2 ( 46.5° ) ⎤⎦
2

2
= 44.1 W.

(d) The rate at which energy is being dissipated by the resistor is

PR = i 2 R = I 2 R sin 2 (ω d t − φ ) = (1.93A ) (16.0 Ω ) sin 2 ⎡⎣( 3000 rad/s )( 0.442 ms ) − 46.5° ⎤⎦


2

= 14.4 W.

(e) Equal. PL + PR + Pc = 44.1W − 17.0 W + 14.4 W = 41.5 W = Pg .


59. We shall use
ε m2 R ε m2 R
Pavg = = .
2Z 2 2 ⎡ R 2 + (ω d L − 1/ ω d C ) ⎤
2
⎣ ⎦

b
where Z = R 2 + ω d L − 1 / ω d C g 2
is the impedance.

(a) Considered as a function of C, Pavg has its largest value when the factor
R 2 + (ωd L − 1/ ωd C ) has the smallest possible value. This occurs for ωd L = 1/ ωd C , or
2

1 1
C= = . × 10−4 F.
= 117
ω L
2
d b2πg b60.0 Hzg c60.0 × 10 Hh
2 2 −3

The circuit is then at resonance.

(b) In this case, we want Z2 to be as large as possible. The impedance becomes large
without bound as C becomes very small. Thus, the smallest average power occurs for C =
0 (which is not very different from a simple open switch).

(c) When ωdL = 1/ωdC, the expression for the average power becomes

ε m2
Pavg = ,
2R

so the maximum average power is in the resonant case and is equal to

( 30.0 V ) = 90.0 W.
2

Pavg =
2 ( 5.00 Ω )

(d) At maximum power, the reactances are equal: XL = XC. The phase angle φ in this case
may be found from
X − XC
tan φ = L = 0,
R
which implies φ = 0° .

(e) At maximum power, the power factor is cos φ = cos 0° = 1.

(f) The minimum average power is Pavg = 0 (as it would be for an open switch).

(g) On the other hand, at minimum power XC ∝ 1/C is infinite, which leads us to set
tan φ = −∞ . In this case, we conclude that φ = –90°.

(h) At minimum power, the power factor is cos φ = cos(–90°) = 0.


60. (a) The power consumed by the light bulb is P = I2R/2. So we must let Pmax/Pmin =
(I/Imin)2 = 5, or

FG I IJ = FG ε IJ = FG Z IJ = FG b g IJ
2
R 2 + ωLmax
2 2 2 2
/ Z min
= 5.
H I K Hε K H Z K GH JK
m max

min m / Z max min R

We solve for Lmax:

= =
2R b
2 120 V / 1000 W g 2

= 7.64 × 10−2 H.
Lmax
ω 2 π 60.0 Hz b g
(b) Yes, one could use a variable resistor.

(c) Now we must let


FG R + Rbulb IJ 2

= 5,
H K
max
Rbulb
or

Rmax = d i
5 − 1 Rbulb = d 5 − 1i
b120 Vg = 17.8 Ω. 2

1000 W

(d) This is not done because the resistors would consume, rather than temporarily store,
electromagnetic energy.
61. (a) The rms current is

ε rms ε rms
I rms = =
Z R 2 + ( 2πfL − 1/ 2πfC )
2

75.0V
=
(15.0 Ω ) { }
+ 2π ( 550Hz )( 25.0mH ) −1/ ⎡⎣ 2π ( 550Hz )( 4.70 µ F ) ⎤⎦
2 2

= 2.59A.

(b) The rms voltage across R is

Vab = I rms R = ( 2.59 A )(15.0 Ω ) = 38.8 V .

(c) The rms voltage across C is

I rms 2.59A
Vbc = I rms X C = = = 159 V .
2πfC 2π ( 550 Hz )( 4.70 µ F )

(d) The rms voltage across L is

Vcd = I rms X L = 2πI rms fL = 2π ( 2.59 A )( 550 Hz )( 25.0 mH ) = 224 V .

(e) The rms voltage across C and L together is

Vbd = Vbc − Vcd = 159.5 V − 223.7 V = 64.2V

(f) The rms voltage across R, C and L together is

Vad = Vab2 + Vbd2 = ( 38.8 V ) + ( 64.2 V ) = 75.0 V


2 2

(g) For R,
V 2 ( 38.8 V )
2

PR = ab = = 100 W.
R 15.0 Ω

(h) No energy dissipation in C.

(i) No energy dissipation in L.


62. For step-up trasnformer:

(a) The smallest value of the ratio Vs / V p is achieved by using T2T3 as primary and T1T3 as
secondary coil: V13/V23 = (800 + 200)/800 = 1.25.

(b) The second smallest value of the ratio Vs / V p is achieved by using T1T2 as primary and
T2T3 as secondary coil: V23/V13 = 800/200 = 4.00.

(c) The largest value of the ratio Vs / V p is achieved by using T1T2 as primary and T1T3 as
secondary coil: V13/V12 = (800 + 200)/200 = 5.00.

For the step-down transformer, we simply exchange the primary and secondary coils in
each of the three cases above.

(d) The smallest value of the ratio Vs / V p is 1/5.00 = 0.200.

(e) The second smallest value of the ratio Vs / V p is 1/4.00 = 0.250.

(f) The largest value of the ratio Vs / V p is 1/1.25 = 0.800.


63. (a) The stepped-down voltage is

F N I = b120 Vg FG 10 IJ = 2.4 V.
Vs = V p GH N JK
s

p
H 500K
Vs 2.4 V
(b) By Ohm’s law, the current in the secondary is I s = = = 016
. A.
Rs 15 Ω

We find the primary current from Eq. 31-80:

F N I = b016
I p = Is GH N JK . Ag FGH 500
s

p
10 I
JK = 3.2 × 10 −3
A.

(c) As shown above, the current in the secondary is I s = 0.16A.


64. We use Eq. 31-79 to find

F N I = b100 Vg FG 500IJ = 100


Vs = V p GH N JK
s

p
H 50 K . × 10 V.
3
c h
65. (a) The rms current in the cable is I rms = P / Vt = 250 × 103 W / 80 × 103 V = 3125
. A.
Therefore, the rms voltage drop is ∆V = I rms R = b3125
. A gb2gb0.30 Ωg = 19
. V.

(b) The rate of energy dissipation is Pd = I rms


2
R = b3125
. A gb2gb0.60 Ωg = 5.9 W.

c h . A , so ∆V = ( 31.25A )( 0.60 Ω ) = 19V.


(c) Now I rms = 250 × 103 W / 8.0 × 103 V = 3125

b
(d) Pd = 3125
. A g b0.60 Ωg = 5.9 × 10 W.
2 2

(e) I rms = 250 × 103 W/ ( 0.80 × 103 V ) = 312.5 A , so ∆V = ( 312.5 A )( 0.60 Ω ) = 1.9 × 102 V .

(f) Pd = ( 312.5 A ) ( 0.60 Ω ) = 5.9 × 104 W.


2
2
66. (a) The effective resistance Reff satisfies I rms Reff = Pmechanical , or

Reff =
Pmechanicalb=
0100
. gb
hp 746 W / hp g
= 177 Ω.
2
I rms b0.650 A
2
g
(b) This is not the same as the resistance R of its coils, but just the effective resistance for
2
power transfer from electrical to mechanical form. In fact I rms R would not give Pmechanical
but rather the rate of energy loss due to thermal dissipation.
67. (a) We consider the following combinations: ∆V12 = V1 – V2, ∆V13 = V1 – V3, and ∆V23
= V2 – V3. For ∆V12,

∆V12 = A sin(ω d t ) − A sin (ω d t − 120° ) = 2 A sin


FG 120°IJ cosFG 2ω t − 120° IJ = b
3 A cos ω d t − 60° g
H 2K H 2 K
d

where we use
sin α − sin β = 2sin ⎡⎣(α − β ) 2 ⎤⎦ cos ⎡⎣(α + β ) 2 ⎤⎦

and sin 60° = 3 2. Similarly,

⎛ 240° ⎞ ⎛ 2ωd t − 240° ⎞


∆V13 = A sin(ωd t ) − A sin (ωd t − 240°) = 2 A sin ⎜ ⎟ cos ⎜ ⎟ = 3 A cos (ωd t − 120° )
⎝ 2 ⎠ ⎝ 2 ⎠
and
⎛ 120° ⎞ ⎛ 2ω d t − 360° ⎞
∆V23 = A sin(ω d t − 120°) − A sin (ω d t − 240°) = 2 A sin ⎜ ⎟ cos ⎜ ⎟
⎝ 2 ⎠ ⎝ 2 ⎠
= 3 A cos (ω d t − 180° )

All three expressions are sinusoidal functions of t with angular frequency ωd.

(b) We note that each of the above expressions has an amplitude of 3A .


68. (a) Eq. 31-39 gives f = ω/2π = (2πCXC)−1 = 8.84 kHz.

(b) Because of its inverse relationship with frequency, then the reactance will go down by
a factor of 2 when f increases by a factor of 2. The answer is XC = 6.00 Ω.
εm 125 V
69. (a) The impedance is Z = = = 39.1 Ω.
I 3.20 A

(b) From VR = IR = ε m cos φ , we get

R=
ε m cos φ
=
b125 Vg cosb0.982 radg = 217. Ω.
I 3.20 A

b g
(c) Since X L − X C ∝ sin φ = sin −0.982 rad , we conclude that XL < XC. The circuit is
predominantly capacitive.
70. (a) Eq. 31-4 directly gives 1/ LC ≈ 5.77×103 rad/s.

(b) Eq. 16-5 then yields T = 2π/ω = 1.09 ms.

(c) Although we do not show the graph here, we describe it: it is a cosine curve with
amplitude 200 µC and period given in part (b).
71. (a) The phase constant is given by

φ = tan −1
FG V − VCIJ FG
V − VL / 2.00
= tan −1 L
IJ
= tan −1 100 b g
H K H . = 45.0° .
K
L
R VL / 2.00

(b) We solve R from ε m cos φ = IR:

R=
ε m cos φ
=
b30.0 Vgbcos 45°g = 70.7 Ω.
I 300 × 10−3 A
72. From Eq. 31-4, we have C = (ω2L)−1 = ((2πf)2L)−1 = 1.59 µF.
73. (a) We solve L from Eq. 31-4, using the fact that ω = 2πf:

1 1
L= = = 6.89 × 10−7 H.
2 2 2
c
4π f C 4π 10.4 × 10 Hz
3
h c340 × 10 Fh
2 −6

(b) The total energy may be calculated from the inductor (when the current is at
maximum):
1
2
1
2
c hc 2
U = LI 2 = 6.89 × 10−7 H 7.20 × 10−3 A = 179h
. × 10−11 J.

(c) We solve for Q from U = 21 Q 2 / C:

c
Q = 2CU = 2 340 × 10−6 F 179 hc
. × 10−11 J = 110 h
. × 10−7 C.
74. (a) With a phase constant of 45º the (net) reactance must equal the resistance in the
circuit, which means the circuit impedance becomes

Z = R 2 ⇒ R = Z/ 2 = 707 Ω.

(b) Since f = 8000 Hz then ωd = 2π(8000) rad/s. The net reactance (which, as observed,
must equal the resistance) is therefore XL – XC = ωdL – (ωdC)−1 = 707 Ω. We are also
told that the resonance frequency is 6000 Hz, which (by Eq. 31-4) means

1 1 1 1
C= = = 2 2 = 2 .
ω L
2
(2π f ) L 4π f L 4π (6000 Hz)2 L
2

Substituting this in for C in our previous expression (for the net reactance) we obtain an
equation that can be solved for the self-inductance. Our result is L = 32.2 mH.

(c) C = ((2π(6000))2L)−1 = 21.9 nF.


75. (a) From Eq. 31-4, we have L = (ω2C)−1 = ((2πf)2C)−1 = 2.41 µH.

(b) The total energy is the maximum energy on either device (see Fig. 31-4). Thus, we
1
have Umax = 2 LI2 = 21.4 pJ.

(c) Of several methods available to do this part, probably the one most “in the spirit” of
this problem (considering the energy that was calculated in part (b)) is to appeal to Umax =
1 2
2 Q /C (from Chapter 26) to find the maximum charge: Q = 2CUmax = 82.2 nC.
76. (a) From Eq. 31-65, we have

FG V − V IJ = tan FG V − (V / 150
. )I
φ = tan −1
H (V / 2.00) JK
−1

H V K
L C L L

R L

which becomes tan–1 (2/3 ) = 33.7° or 0.588 rad.

(b) Since φ > 0, it is inductive (XL > XC).

(c) We have VR = IR = 9.98 V, so that VL = 2.00VR = 20.0 V and VC = VL/1.50 = 13.3 V.


Therefore, from Eq. 31-60, we have

ε m = VR2 + (VL − VC ) 2 = (9.98 V) 2 + (20.0 V − 13.3 V) 2 = 12.0 V .


77. (a) The impedance is Z = (80.0 V)/(1.25 A) = 64.0 Ω.

(b) We can write cos φ = R/Z ⇒ R = (64.0 Ω)cos(0.650 rad) = 50.9 Ω.

(c) Since the “current leads the emf” the circuit is capacitive.
78. (a) We find L from X L = ωL = 2πfL:

XL . × 103 Ω
130
f = = = 4.60 × 103 Hz.
c
2 πL 2 π 45.0 × 10 H
−3
h
(b) The capacitance is found from XC = (ωC)–1 = (2πfC)–1:

1 1
C= = = 2.66 × 10−8 F.
c hc
2 πfX C 2 π 4.60 × 10 Hz 130
3
. × 10 Ω
3
h
(c) Noting that XL ∝ f and XC ∝ f –1, we conclude that when f is doubled, XL doubles and
XC reduces by half. Thus, XL = 2(1.30 × 103 Ω) = 2.60 × 103 Ω .

(d) XC = 1.30 × 103 Ω/2 = 6.50 × 102 Ω.


79. (a) Using ω = 2πf , XL = ωL, XC = 1/ωC and tan(φ) = (XL −XC)/R, we find

φ = tan−1[(16.022 – 33.157)/40.0] = –0.40473 ≈ –0.405 rad.

(b) Eq. 31-63 gives I = 120/ 402 + (16-33)2 = 2.7576 ≈ 2.76 A.

(c) XC > XL ⇒ capacitive.


1 2
80. From Umax = 2 LI we get I = 0.115 A.
81. From Eq. 31-4 we get f = 1/2π LC = 1.84 kHz
82. (a) The reactances are as follows:

X L = 2π f d L = 2π (400 Hz)(0.0242 H) = 60.82 Ω


X C = (2π f d C ) −1 = [2π (400 Hz)(1.21×10−5 F)]−1 = 32.88 Ω

Z = R 2 + ( X L − X C ) 2 = (20.0 Ω) 2 + (60.82 Ω − 32.88 Ω) 2 = 34.36 Ω

With ε = 90.0 V, we have

ε 90.0 V I 2.62 A
I= = = 2.62 A ⇒ I rms = = = 1.85 A .
Z 34.36 Ω 2 2

Therefore, the rms potential difference across the resistor is VR rms = Irms R = 37.0 V.

(b) Across the capacitor, the rms potential difference is VC rms = Irms XC = 60.9 V.

(c) Similarly, across the inductor, the rms potential difference is VL rms = Irms XL = 113 V.

(d) The average rate of energy dissipation is Pavg = (Irms)2R = 68.6 W.


83. (a) At any time, the total energy U in the circuit is the sum of the energy UE in the
capacitor and the energy UB in the inductor. When UE = 0.500UB (at time t), then UB =
2.00UE and U = UE + UB = 3.00UE. Now, UE is given by q 2 / 2C , where q is the charge
on the capacitor at time t. The total energy U is given by Q 2 / 2C , where Q is the
maximum charge on the capacitor. Thus,

Q 2 3.00q 2 Q
= ⇒ q= = 0.577Q .
2C 2C 3.00

(b) If the capacitor is fully charged at time t = 0, then the time-dependent charge on the
capacitor is given by q = Q cos ωt . This implies that the condition q = 0.577Q is satisfied
when cosωt =0.557, or ωt = 0.955 rad. Since ω = 2π / T (where T is the period of
oscillation), t = 0.955T / 2 π = 0.152T , or t / T = 0.152.
84. From Eq. 31-60, we have (220 V / 3.00 A) 2 = R 2 + X L2 ⇒ X L = 69.3 Ω .
85. (a) The energy stored in the capacitor is given by U E = q 2 / 2C. Since q is a periodic
function of t with period T, so must be UE. Consequently, UE will not be changed over
one complete cycle. Actually, UE has period T/2, which does not alter our conclusion.

(b) Similarly, the energy stored in the inductor is U B = 21 i 2 L . Since i is a periodic


function of t with period T, so must be UB.

(c) The energy supplied by the generator is

b g FGH 21 T IJK ε
Pavg T = I rmsε rms cos φ T = m I cos φ

where we substitute I rms = I / 2 and ε rms = ε m / 2 .

(d) The energy dissipated by the resistor is

b g b g FGH 21 T IJK I R.
Pavg,resistor T = I rmsVR T = I rms I rms R T = 2

b g b g
(e) Since ε m I cos φ = ε m I VR / ε m = ε m I IR / ε m = I 2 R, the two quantities are indeed the
same.
86. (a) We note that we obtain the maximum value in Eq. 31-28 when we set

π 1 1
t= = = = 0.00417 s
2ω d 4 f 4(60)

or 4.17 ms. The result is ε m sin( π / 2) = ε m sin (90° ) = 36.0 V .

(b) At t = 4.17 ms, the current is

i = I sin (ω d t − φ ) = I sin (90° − (−24.3°)) = (0.164A) cos(24.3°) = 0.1495A ≈ 0.150 A.

using Eq. 31-29 and the results of the Sample Problem. Ohm’s law directly gives

vR = iR = (0.1495A)(200Ω) = 29.9V.

(c) The capacitor voltage phasor is 90° less than that of the current. Thus, at t = 4.17 ms,
we obtain

vC = I sin(90° − (−24.3°) − 90°) X C = IX C sin(24.3°) = (0.164A)(177Ω) sin(24.3°) = 11.9V.

(d) The inductor voltage phasor is 90° more than that of the current. Therefore, at t =
4.17 ms, we find

vL = I sin(90° − (−24.3°) + 90°) X L = − IX L sin(24.3°) = −(0.164A)(86.7Ω) sin(24.3°)


= −5.85V.

(e) Our results for parts (b), (c) and (d) add to give 36.0 V, the same as the answer for
part (a).
b g
87. (a) Let ωt − π / 4 = π / 2 to obtain t = 3π / 4ω = 3π / 4 350 rad / s = 6.73 × 10−3 s.

b g
(b) Let ωt + π / 4 = π / 2 to obtain t = π / 4ω = π / 4 350 rad / s = 2.24 × 10−3 s.

(c) Since i leads ε in phase by π/2, the element must be a capacitor.

(d) We solve C from X C = ωCb g −1


= εm / I :

I 6.20 ×10−3 A
C= = = 5.90 × 10−5 F.
ε mω ( 30.0 V )( 350 rad/s )
88. (a) The amplifier is connected across the primary windings of a transformer and the
resistor R is connected across the secondary windings.

(b) If Is is the rms current in the secondary coil then the average power delivered to R is
Pavg = I s2 R . Using I s = ( N p / N s ) I p , we obtain

F I N IJ
=G
p p
2

Pavg
HN K s
R.

Next, we find the current in the primary circuit. This is effectively a circuit consisting of
a generator and two resistors in series. One resistance is that of the amplifier (r), and the
other is the equivalent resistance Req of the secondary circuit. Therefore,

ε rms ε rms
Ip = =
r + Req d
r + N p / Ns R i 2

where Eq. 31-82 is used for Req. Consequently,

ε 2 ( N p / N s )2 R
Pavg = 2
.
⎡⎣ r + ( N p / N s ) 2 R ⎤⎦

Now, we wish to find the value of Np/Ns such that Pavg is a maximum. For brevity, let x =
(Np/Ns)2. Then
ε 2 Rx
Pavg =
b
r + xR
2
,
g
and the derivative with respect to x is

dPavg
=
b
ε 2 R r − xR g.
dx br + xRg 3

This is zero for x = r / R = b1000 Ωg / b10 Ωg = 100 . We note that for small x, P avg
increases linearly with x, and for large x it decreases in proportion to 1/x. Thus x = r/R is
indeed a maximum, not a minimum. Recalling x = (Np/Ns)2, we conclude that the
maximum power is achieved for
N p / N s = x = 10 .
The diagram that follows is a schematic of a transformer with a ten to one turns ratio. An
actual transformer would have many more turns in both the primary and secondary coils.
89. Resonance occurs when the inductive reactance equals the capacitive reactance.
Reactances of a certain type add (in series) just like resistances did in Chapter 28. Thus,
since the resonance ω values are the same for both circuits, we have for each circuit:

1 1
ω L1 = , ω L2 =
ωC1 ω C2

and adding these equations we find


1⎛ 1 1 ⎞
ω ( L1 + L2 ) = ⎜ + ⎟.
ω ⎝ C1 C2 ⎠

Since Leq = L1 + L2 and Ceq−1 = (C1−1 + C2−1 ) ,

1
ω Leq = ⇒ resonance in the combined circuit.
ω Ceq
90. When switch S1 is closed and the others are open, the inductor is essentially out of the
circuit and what remains is an RC circuit. The time constant is τC = RC. When switch S2
is closed and the others are open, the capacitor is essentially out of the circuit. In this case,
what we have is an LR circuit with time constant τL = L/R. Finally, when switch S3 is
closed and the others are open, the resistor is essentially out of the circuit and what
remains is an LC circuit that oscillates with period T = 2π LC . Substituting L = RτL and
C = τC/R, we obtain T = 2π τ Cτ L .
91. When the switch is open, we have a series LRC circuit involving just the one
capacitor near the upper right corner. Eq. 31-65 leads to

1
ωd L −
ωd C
= tan φo = tan(−20°) = − tan 20°.
R

Now, when the switch is in position 1, the equivalent capacitance in the circuit is 2C. In
this case, we have
1
ωd L −
2ω d C
= tan φ1 = tan10.0°.
R

Finally, with the switch in position 2, the circuit is simply an LC circuit with current
amplitude
εm εm εm
I2 = = =
Z LC 2 1 −ω L
⎛ 1 ⎞ ωd C
⎜ωd L − ω C ⎟
d

⎝ d ⎠

where we use the fact that (ω d C ) −1 > ω d L in simplifying the square root (this fact is
evident from the description of the first situation, when the switch was open). We solve
for L, R and C from the three equations above, and the results are

−ε m −120V
(a) R = = = 165 Ω.
I 2 tan φo (2.00 A) tan (−20.0°)

εm ⎛ tan φ1 ⎞ 120 V ⎛ tan10.0° ⎞


(b) L = ⎜1 − 2 ⎟= ⎜1 − 2 ⎟ = 0.313 H .
ωd I 2 ⎝ tan φo ⎠ 2π(60.0 Hz)(2.00 A) ⎝ tan (−20.0°) ⎠

(c)
I2 2.00 A
C= =
2ωd ε m (1 − tan φ1 / tan φ0 ) 2(2π)(60.0 Hz)(120 V) (1 − tan10.0° / tan(−20.0°) )
= 1.49 ×10−5 F
1
92. (a) Eqs. 31-4 and 31-14 lead to Q = = I LC = 1.27 × 10−6 C .
ω

(b) We choose the phase constant in Eq. 31-12 to be φ = −π / 2 , so that i0 = I in Eq.


31-15). Thus, the energy in the capacitor is

q 2 Q2
UE = = (sin ωt ) 2 .
2C 2C

Differentiating and using the fact that 2 sin θ cos θ = sin 2θ, we obtain

dU E Q 2
= ω sin 2ωt .
dt 2C

We find the maximum value occurs whenever sin 2ωt = 1 , which leads (with n = odd
integer) to
1 nπ nπ nπ
t= = = LC = 8.31 × 10−5 s, 2.49 × 10−4 s,… .
2ω 2 4ω 4

The earliest time is t = 8.31× 10−5 s.

(c) Returning to the above expression for dU E / dt with the requirement that sin 2ωt = 1 ,
we obtain

FG dU IJ Q2
ω=
dI LC i 2
I I2 L
= 5.44 × 10−3 J / s .
H dt K = =
E

max 2C 2C LC 2 C
93. (a) We observe that ω = 6597 rad/s, and, consequently, XL = 594 Ω and XC = 303 Ω.
Since XL > XC, the phase angle is positive: φ = +60.0° .

X L − XC
From Eq. 31-65, we obtain R = = 168Ω .
tan φ

(b) Since we are already on the “high side” of resonance, increasing f will only decrease
the current further, but decreasing f brings us closer to resonance and, consequently, large
values of I.

(c) Increasing L increases XL, but we already have XL > XC. Thus, if we wish to move
closer to resonance (where XL must equal XC), we need to decrease the value of L.

(d) To change the present condition of XC < XL to something closer to XC = XL (resonance,


large current), we can increase XC. Since XC depends inversely on C, this means
decreasing C.
94. (a) We observe that ωd = 12566 rad/s. Consequently, XL = 754 Ω and XC = 199 Ω.
Hence, Eq. 31-65 gives
FG
X − XC
φ = tan −1 L
IJ
H R K
= 122
. rad .

εm
(b) We find the current amplitude from Eq. 31-60: I = = 0.288 A .
R2 + ( X L − X C )2
95. From Eq. 31-4, with ω = 2 πf = 4.49 × 103 rad / s, we obtain

1
L= = 7.08 × 10−3 H.
ω 2C
96. (a) From Eq. 31-4, with ω = 2πf , C = 2.00 nF and L = 2.00 mH, we have

1
f = = 7.96 × 104 Hz.
2π LC

VC
(b) The maximum current in the oscillator is imax = I C = = ωCvmax = 4.00 × 10 −3 A.
XC

(c) Using Eq. 30-49, we find the maximum magnetic energy:

1 2
U B ,max = Limax = 1.60 × 10−8 J.
2

(d) Adapting Eq. 30-35 to the notation of this chapter, vmax = L | di / dt |max , which yields a
(maximum) time rate of change (for i) equal to 2.00×103 A/s.
97. Reading carefully, we note that the driving frequency of the source is permanently set
at the resonance frequency of the initial circuit (with switches open); it is set at ωd =
1/ LC = 1.58 × 104 rad/s and does not correspond to the resonance frequency once the
switches are closed. In our table, below, Ceq is in µF, f is in kHz, and Req and Z are in Ω.
Steady state conditions are assumed in calculating the current amplitude (which is in
amperes); this I is the current through the source (or through the inductor), as opposed to
the (generally smaller) current in one of the resistors. Resonant frequencies f are
computed with ω = 2πf. Reducing capacitor and resistor combinations is explained in
chapters 26 and 28, respectively.

(a) (b) (c) (d) (e)


switch Ceq(µF) f(kHz) Req(Ω) Z(Ω) I (A)
S1 4.00 1.78 12.0 19.8 0.605
S2 5.00 1.59 12.0 22.4 0.535
S3 5.00 1.59 6.0 19.9 0.603
S4 5.00 1.59 4.0 19.4 0.619

Potrebbero piacerti anche